LSAT and Law School Admissions Forum

Get expert LSAT preparation and law school admissions advice from PowerScore Test Preparation.

User avatar
 Dave Killoran
PowerScore Staff
  • PowerScore Staff
  • Posts: 5853
  • Joined: Mar 25, 2011
|
#80121
Complete Question Explanation
(The complete setup for this game can be found here: lsat/viewtopic.php?t=6605)

The correct answer choice is (A)

Answer choice (A) is the correct answer. If R and S form an RS block, then from the second rule,
     S must run in the third race and R must run in the second race. That enacts the fourth rule,
     eliminating U from running in the track meet. Thus, Q and T must be the other two runners in
     the meet, and from the first rule they form a QT block. But, with R and S running in the second
     and third races, there is no room for the QT block. Thus, this answer choice cannot occur and is
     correct.

Answer choice (B) can be eliminated by this hypothetical: S-Q-T-U.

Answer choice (C) can be easily eliminated by referring to the solution to question #12: R-U-S-T.

Answer choice (D) can be eliminated by this hypothetical: U-Q-T-R.

Answer choice (E) can be eliminated by this hypothetical: U-T-S-R.
 Tajadas
  • Posts: 63
  • Joined: Apr 11, 2020
|
#80102
Hi, I got A correct, but only because I started doing process of elimination starting with A. If the correct answer choice had been one of the last choices presented, I would've spent 2-3 minutes on this problem, way more than I could ever have time for. Do you have any suggestions on how to do these kinds of questions as quickly as possible?
 Jeremy Press
PowerScore Staff
  • PowerScore Staff
  • Posts: 1000
  • Joined: Jun 12, 2017
|
#80119
Hi Tajadas,

Your best bet on a Global-CANNOT question, if nothing jumps out after a quick read of all four answer choices, is to continue with the game and come back to the question after you've diagrammed as many legitimate solutions as you need to for local questions in the game. Use those local-question solutions to eliminate answer choices. For example, answer choice D to question number 12 shows that S can run in the race immediately before T, which eliminates answer choice C on this question. Similarly, you can use your work for questions 16 and 17 to eliminate other answers on this question. If my memory serves on this game, there are enough diagrammed solutions on the other questions to eliminate every wrong answer on this question.

After the fact, though, when you're in review mode, try to determine whether you could've predicted the answer using the restrictions on variables that you know exist in the game. Here, an interestingly restricted answer is answer choice A. For anything to be immediately before Smith, it has to be in position 2 (since Smith can only go in 1 or 3). So for A to work, you must put R in position 2, which immediately triggers another rule in the game (it forces U out). Now all that's left are Q and T, which can't be split when they are in.

No other answer is so specific about the exact placement of the variables involved! For S to be immediately before something else, it could still be in 1 or in 3 (so answers B and C aren't as immediately restrictive on the placement of the variables involved). We similarly can't tell what's going on with the placement of variables in answers D and E (since T isn't that restricted until we know Q is in), so there are likely to be more options for those variables. You can use some of these intuitions about the variables and their placements to try to determine the best answer to begin testing. You might not be perfect about this at first, but the more you try, the better you'll get at it.

I hope this helps!

Get the most out of your LSAT Prep Plus subscription.

Analyze and track your performance with our Testing and Analytics Package.